0 Daumen
182 Aufrufe

Aufgabe:

Zeigen sie für n ∈ ℕ gilt \(\sum_{k=0}^{n} \frac{1}{k^2} ≤ 2 - \frac{1}{n}\) )

Problem/Ansatz:

Es wird zwar nicht direkt vorgeschrieben aber wir haben sonst meistens die Vollständige Induktion genutzt.

Den Induktions-Anfang hab ich ohne Probleme lösen können.

Als Induktions-Voraussetzung (IV) hab ich:

\(\sum_{k=1}^{n+1} \frac{1}{k^2} ≤ 2 - \frac{1}{(n+1)}\)

Probleme hab ich beim Induktions-Schritt. Mein Ansatz ist:

\(\sum_{k=1}^{n+1} \frac{1}{k^2}  = \sum_{k=1}^{n} + \frac{1}{(n+1)^2} ≤ 2 - \frac{1}{n} + \frac{1}{(n+1)^2}\) (nach IV)

Soweit ich weiß muss ich jetzt die rechte Seite der letzten Ungleichung so umformen das die rechte Seite der IV raus kommt. Komme aber leider nicht drauf wie ich das anstelle.

Avatar von

1 Antwort

0 Daumen
 
Beste Antwort

Induktionsschritt n → n + 1

Σ (k = 1 bis n + 1) (1/k^2) ≤ 2 - 1/(n + 1)
Σ (k = 1 bis n) (1/k^2) + (1/(n + 1)^2) ≤ 2 - 1/(n + 1)
2 - 1/n + (1/(n + 1)^2) ≤ 2 - 1/(n + 1)
- 1/n + (1/(n + 1)^2) ≤ - 1/(n + 1)
- (n + 1)^2/n + 1 ≤ - (n + 1)
-n - 1 - 1/n ≤ - n - 1
- 1/n ≤ 0 → stimmt

Avatar von 480 k 🚀

Vielen Dank :)

Ein anderes Problem?

Stell deine Frage

Willkommen bei der Mathelounge! Stell deine Frage einfach und kostenlos

x
Made by a lovely community